LSAT and Law School Admissions Forum

Get expert LSAT preparation and law school admissions advice from PowerScore Test Preparation.

 Administrator
PowerScore Staff
  • PowerScore Staff
  • Posts: 8915
  • Joined: Feb 02, 2011
|
#85572
Complete Question Explanation

The correct answer choice is (C).

Answer choice (A):

Answer choice (B):

Answer choice (C): This is the correct answer choice.

Answer choice (D):

Answer choice (E):

This explanation is still in progress. Please post any questions below!
User avatar
 ashpine17
  • Posts: 321
  • Joined: Apr 06, 2021
|
#98659
is this a must be false?
 Luke Haqq
PowerScore Staff
  • PowerScore Staff
  • Posts: 751
  • Joined: Apr 26, 2012
|
#99365
Hi ashpine17!

Yes, this is a cannot be true/must be false question. With that type of question, four of the answer choices could be true or must be true, while the correct answer choice cannot be true/must be false.
 elishahasan18@gmail.com
  • Posts: 1
  • Joined: Aug 05, 2023
|
#105170
Hi I am just confused as to why the answer is C and not E?
 Robert Carroll
PowerScore Staff
  • PowerScore Staff
  • Posts: 1787
  • Joined: Dec 06, 2013
|
#105226
elishahasan18,

Answer choice (E) seems extremely well-supported by the third paragraph: "Mass and Portman found that minor
eruptions have no discernible effect on temperature." So, minor eruptions have no discernible effect on temperature. Answer choice (E) is a specific instance of that general claim, so seems true. That's not at all what we're looking for for this Cannot Be True question.

Answer choice (C) contradicts the rest of that paragraph, which is why it's the right answer.

Robert Carroll

Get the most out of your LSAT Prep Plus subscription.

Analyze and track your performance with our Testing and Analytics Package.